LSAT and Law School Admissions Forum

Get expert LSAT preparation and law school admissions advice from PowerScore Test Preparation.

 Administrator
PowerScore Staff
  • PowerScore Staff
  • Posts: 8917
  • Joined: Feb 02, 2011
|
#81330
Complete Question Explanation

Weaken. The correct answer choice is (E).

Answer choice (A):

Answer choice (B):

Answer choice (C):

Answer choice (D):

Answer choice (E): This is the correct answer choice.

This explanation is still in progress. Please post any questions below!
 MannyH
  • Posts: 10
  • Joined: Jun 19, 2018
|
#57161
I could not find an explanation for the this particular question. I selected C with E also as a contender which was the correct answer.
 James Finch
PowerScore Staff
  • PowerScore Staff
  • Posts: 943
  • Joined: Sep 06, 2017
|
#57687
Hi Manny,

The stimulus here is similar to an Assumption question, except the stem is asking for why it is flawed as opposed to what is necessary for the argument. Either way, there is an assumption that isn't stated in the stimulus: that while a single business or university can't create the proposed network by itself, these non-governmental entities could collaborate together to fund the network. So that's what I prephrased, and what I looked for in the answer choices.

Answer choice (C) deals with something that isn't a part of the stimulus (maintenance) and isn't relevant to it. Ultimately, we need to know who can fund the initial setup of the network, not who maintains it.

Answer choice (E) is correct because it fits exactly with our prephrase, as the missing supporter assumption necessary to make the argument work. If (E) is true, then the conclusion, that the government needs to fund the network, would not be true either.

Hope this helps!
 syl4655
  • Posts: 1
  • Joined: Sep 27, 2018
|
#58581
Hello,

I was able to eliminate the choices down to B and E, since both seem to point to other ways the supercomputer network can be funded, either by a third party or both government and industry/universities. Could someone explain why E is a better answer than B?
 James Finch
PowerScore Staff
  • PowerScore Staff
  • Posts: 943
  • Joined: Sep 06, 2017
|
#58753
Hi SYL,

National versus international concerns isn't something mentioned by the stimulus and requires one or more assumptions to be relevant to the argument being made. Whether the network is international or not, we're still left with the question of who would fund it: would it be a single business or university, or a government? And that gets us to the heart of the issue, in that the stimulus has created a false dilemma with the way it is worded: either a single business or university must fund the network, or a single government must do so. But obviously this need not be true, and multiple funding sources could cooperate. That's the false dilemma being created, not whether the network is international or not.

Hope this helps!
 Jude.m.stone@gmail.com
  • Posts: 18
  • Joined: Mar 12, 2023
|
#100756
Hello, I can't figure out why A can't be a contender. Can you please explain how you eliminated it?

Also, regarding E being correct, I had eliminated that answer choice because Option E says "It overlooks the possibility that businesses or universities, or both, could cooperate to build the network," but the stimulus says "...and no business or university wants to invest in a part of a network if no mechanism exists for coordinating establishment of the network as a whole." The stimulus's language that no business or university WANTS to coordinate the network's establishment counters Option E's language in my opinion -- the stimulus didn't say they COULDN'T coordinate, it said they don't want to. To me, that doesn't imply that the stimulus overlooked the possibility of coordinating. Can you please explain why my reasoning is wrong? Thanks for your help!
User avatar
 Paul Popa
PowerScore Staff
  • PowerScore Staff
  • Posts: 64
  • Joined: Sep 20, 2022
|
#100763
Hey there Jude,

I'd be happy to help! Regarding (A), I would say the author does in fact provide a resolution to the dilemma (if we consider the inability of a single university or business to purchase a supercomputer network to be a dilemma): the government! The government should provide the funds to put the network in place. It's also important to remember that an author never has to provide a resolution to a problem. If the author's argument is that we shouldn't choose a certain option, but doesn't say what option we should take, that isn't necessarily a flaw; we have to evaluate the author's argument on its own grounds, which means deciding whether their evidence is sufficient for their conclusion.

Careful with (E)--the argument says that no business or university wants to invest in a part of a network if no mechanism exists for coordinating establishment of the network as a whole. But, if universities can cooperate with other universities (and businesses with other businesses) suddenly there is now a mechanism to establish the network as a whole: if we work together! The argument is saying that no university or business would agree to creating only part of the network, but this doesn't mean they're against the idea altogether. They just need a way to get a complete network created. Hope this helps!

Get the most out of your LSAT Prep Plus subscription.

Analyze and track your performance with our Testing and Analytics Package.